Jump to content
Urch Forums

Dahlia

Members
  • Posts

    35
  • Joined

Converted

  • My Tests
    No

Dahlia's Achievements

Newbie

Newbie (1/14)

9

Reputation

  1. thanks a lot dear, but seems that I didnt recognize the problem right too..:S
  2. well bscout, I am afriad zero is not the answer, as it's not there on the answer choices, as far as I remember some of the choices were:1/710 under root7/10 all under root
  3. Guys,great job , I had this one on one of the exams I've takenA set of numbers have a standard deviation 0, if each of those numbers in the set was increased by 10, then each of them was divided by 7, what would be thier standard deviation?
  4. Well comp, If n is somewhere on the line between 4 and 12, and u can get n by either ways n=4+2x OR n=12-x where 2x is the distance from n to 4 as we know ,and x is the distance from n to 12 got it comp?
  5. Hey Comp, n is 2 times far from 4 as from 12, i.e. if n is x far from 12 then it's 2x far from 4 Got it now?
  6. 3)If n is the number on the number line between 4 and 12 thats is twice as far from 4 as from 12 then n is 6*(2/3) 8 9*(1/3) 10*(1/2) 11*(1/4) Answer: # draw a line and place 4 on the left part of it and 12 on the right part of the line, place n somewhere on the line closer to 12 as the problem says that n is twice as far from 4 as from 12. # If X is the distance of n from 4 and 12, therefore n is 2x far from 4 and x far from 12 ( n is twice as far from 4 as from 12) # Then the distance from 4 to 12 =2x+x=3x, and since we have 8 numbers between 4 and 12, therefore 3x=8, and X=8/3 # Since n is 2x far from 4, therefore n= 4+2x = 4+ 2(8/3)= 4+ 16/3 = 9 1/3 or 28/3 . The answer is C 5) The sum of x distinct integers greater than zero is less than 75 . what is the greatest value of x 8 9 10 11 12 Answer: We know that any integer equals the summation of the previous integers, e.g 2 = 1+2 which is 3 then 3 = 3+ (3) where (3) is the summation of 1+2 and so on..so if we took the numbers from 1 to 12 (since 12 is the greatest number in the given choices), then for 1.....1 for 2.....1+2=3 for 3.....3+3=6 for 4.....6+4=10 for 5.....10+5=15 for 6.....15+6=21 for 7.....21+7=28 for 8.....28+8=36 for 9.....36+9=45 for 10...45+10=55 for 11...55+11=66 for 12...66+12=78 which is > 75 therefore 11 is the greatest value of X The answer is D Hope it's clear...
  7. thnx guys :) this was so clear
  8. 1) If n is an integer and 99 2 4 6 8 10 2) n= k+(r/s)/(t/v), if all of the values are positive, which one of them when multiplied by 2 will reduce the value of n to 1/2 its present value? k r s t v 3) A gram of carbohydrates has 4 calories, a gram of fat has 9 calories compare Column A: number of grams of carbohydrates that has x calories Column B: number of grams of fat that has x calories 4) Jing takes x hours to mow certain large lawn and bill takes twice as long as jing takes to mow the lawn. compare Column A: number of hours it takes Jing and Bill working together at thier own rates to mow the lawn Coulmn B: 3x/4 5) The average (arithmatic mean) height for a group of m men is 70 inch , and avergae height of f women group is 64 inch. When groups are combined together, the avergae height is 66 inch. Column A: m Column B: f Can anyone give answers to those please...explain how did u get the answer cuz I couldnt get them :hmm:
  9. Dahlia

    Solve!!

    Hey Arun:) here you are the.. first we have a given mRn, where m=n^2-1, so if we considered the same for rRs and sRp we'll have r=s^2-1 and s=p^2-1 subsitue with the value of s in the r equation... u get r= (p^2-1)^2 -1 Did u get it dude?
  10. riteshen, u cant compare it to this example of urs cuz the problem is saying that each of them had a part of a whole. so if John had 3/7th and Wren had 4/14 first substract Wren's share from John's share 3/7-5/14 = 6-5/14 = 1/14 ( this is the difference which gives the exceeded amount) we want to get how many times is John's of Wren's share. Divide the exceeded amount we got by Wren's share====> (1/14)/(5/14)====> 1/5 Is it clear now?
  11. Dahlia

    Solve!!

    just a lil bit of concenration and they'd be lucky to have u there man:)
  12. Dahlia

    Solve!!

    Area of a circle is Pi r^2 so for column A: radius=r ==> 4 pi r^2 for coulmn B: radius=2r ====> pi (2r)^2 ===> 4 pi r^2 therefore answer is C as for circular mats and circular place mats..both tell that it has a the shape of a circle and thus the area of a circle! did u get it?
  13. Okay now..I need someone to explain this to me as if I am 3 yrs old baby gurl!!
  14. Dahlia

    Solve!!

    Thnx bscout for the illustration... this was very helpful and clear
  15. Dahlia

    Solve!!

    1) If the assets of a certain mutual fund dropped by $3.04 billions to 72.04 billions. What's the approximate percent decrease in the fund's assets? 0.4% 4% 5% 25% 40% 2) The total cost for a certain company to produce a newline of hats is $15000 for designs and $5 per hat for the manufacturing costs. The revenue rom the sale of each hat is $9 In order for the total sales revenue to be equal to the total production cost m hats must be produced and sold. Compare coulmn A: m column B: 4000 3) Compare Column A: the maximum surface area that can be covered by 4 circular place mats each of radius r. coulmn B: the maximum surface area that can be covered by 1 circular mat of radius 2r. answer: C 4) The expression mRn, where m and n are positive integers denotes that m= n^2-1. If rRs and sRp then in terms of p, r equals. p^4 p^4-1 (p^2-1)^2 (p^2-1)^2-1 (p-1)^4 5) For every cubic foot of coal burned, a chimney expels x cubic feet of smoke. If smoke was expelled at the rate of y cubic feet per minute for w minutes. which of the following expresses the number of cubic feet of coal that's burned during w minutes? xy/w xw/y yw/x x/w+y y/x+w Please illustrate ur answers...thnx
×
×
  • Create New...